El resultado experimental no se puede explicar mediante la teoría para un sistema de 2 resortes y 1 masa

Tenemos 2 resortes 1 sistema de masa en 2D como se muestra,

Sistema

Aquí está mi breve intento de solución:

F X = F 1 X + F 2 X = k ( X + yo ) i ^ k ( X yo ) i ^ = 2 k X i ^ X ¨ + 2 k metro X = 0 ,
F y = F 1 y + F 2 y = 2 k y ȷ ^ y ¨ + 2 k metro y = 0.
La solución general para estas ecuaciones es;
X ( t ) = A pecado ( ω t ) + B porque ( ω t ) ,
y ( t ) = C pecado ( ω t ) + D porque ( ω t ) ,
dónde ω = 2 k metro . Evaluando las condiciones iniciales de la siguiente manera;
X ( 0 ) = X 0 X ( 0 ) = A pecado ( 0 ) + B porque ( 0 ) = B = X 0 ,
y ( 0 ) = y 0 y ( 0 ) = C pecado ( 0 ) + D porque ( 0 ) = D = y 0 ,
X ˙ ( 0 ) = V 0 X = 0 X ˙ ( 0 ) = A ω porque ( 0 ) X 0 ω pecado ( 0 ) = A = 0 ,
y ˙ ( 0 ) = V 0 y = 0 y ˙ ( 0 ) = C ω porque ( 0 ) y 0 ω pecado ( 0 ) = C = 0 ,
X ( t ) = X 0 porque ( ω t ) , y ( t ) = y 0 porque ( ω t ) .
Verifiqué esta solución con otro método dado aquí en la primera respuesta y son consistentes. Observe que hay un pequeño error en la última ecuación, debería ser metro en lugar de 2 metro ; puede verificar aquí en la primera respuesta.

Hice una figura de esta solución y aquí está:

ingrese la descripción de la imagen aquí

En la facultad, hemos realizado este experimento y el resultado se parece a esto (también hecho por mí):

ingrese la descripción de la imagen aquí

Los puntos muestran la posición de la masa. La única diferencia entre estas dos imágenes es el cambio de fase. Para obtener la figura experimental agrego un cambio de fase de a y ( t ) y;

y ( t ) = y 0 porque ( ω t + ϕ ) , ϕ = arcán ( y 0 / X 0 ) .

Y también está esto: cuando realizamos este experimento en el laboratorio, el instructor dijo que el X ( t ) y y ( t ) debe tener un cambio de fase de π / 2 , escribirse entre sí, es decir, si X ( t ) porque ( ω t ) entonces y pecado ( ω t ) y viceversa. Y este fue el caso real en el laboratorio.

Mi pregunta es, ¿cómo puedo obtener este cambio de fase de las ecuaciones -legalmente-? ¿O hay alguna explicación?

Editar:

Es un 50 C metro × 50 C metro X y plano horizontal, por lo que no se aplica g en el sistema. metro = 570 gramo r y k 60000 d y norte C metro . La longitud en reposo de los resortes es yo 0 = 13 C metro . Para realizar el experimento, primero estiramos ambos resortes y los unimos a la masa. El nuevo equilibrio ocurre cuando la longitud de los resortes es aproximadamente 25 C metro . Creo que este es un estiramiento bastante grande, pero que yo sepa, la elasticidad no se rompe.

Aquí hay un video corto de modos normales y pequeñas oscilaciones: https://www.youtube.com/watch?v=eyEpFeZO9W8 En el laboratorio, configuramos este experimento con amplitudes mucho mayores en ambas direcciones. Proporcionaré algunas fotos y datos reales tan pronto como pueda.

+1 por esfuerzo e investigación. No hay gravedad en su análisis. ¿Estaban las oscilaciones confinadas a un plano horizontal? ¿O los resortes estaban tan tensos que se puede despreciar la gravedad?
¿Qué hay en los ejes de sus parcelas? ¿Son y(t) frente a x(t)?
Las condiciones iniciales dictan si la oscilación para el X y y los ejes van a estar en fase o no. Creo que el experimento y la teoría usan diferentes condiciones iniciales.
¿Puede proporcionar más detalles sobre su experimento? ¿Cuál es la longitud de reposo y la longitud de equilibrio de cada resorte? ¿Cuáles son los valores de k y metro ? Es el y dirección vertical, o es el X y plano horizontal? ¿Cómo se inicia la oscilación del sistema? ¿Cuáles son sus resultados reales? (Supongo que su último gráfico es una ilustración y no datos reales).
Es un 50 C metro × 50 C metro X y plano horizontal, por lo que no se aplica g en el sistema. metro = 570 gramo r y k 60000 d y norte C metro . La longitud en reposo de los resortes es yo 0 = 13 C metro . Y sí, he hecho las tramas en esta pregunta. los ejes son X y . Pero puedo agregar datos reales mañana, después de clase.
Sí, sus datos reales serían muy útiles. ¿Hiciste un video de las oscilaciones?
Tengo un buen video de los modos normales del sistema y una oscilación realmente pequeña, intentaré agregar este video aquí en solo un minuto. Y proporcionaré los datos reales mañana, también el video completo -si puedo agregar un video aquí-
@ Saba: ¿Hay alguna diferencia entre estirar primero los resortes y luego conectarlos a la masa y estirar los resortes con la masa ya unida?
@Saba ¿Puede proporcionar datos de su experimento? por ejemplo, tablas de x(t) e y(t). ¿También tiene datos de su medición de la constante de resorte?

Respuestas (3)

Tus ecuaciones de movimiento son incorrectas. Para entender por qué, considere el caso en esta imagen:

ingrese la descripción de la imagen aquí

¿Cuáles son los X y y componentes de la fuerza F actuando sobre la masa?

Si la longitud restante del resorte es yo 0 y su constante elástica es k , la fuerza F es

F = k r ^ ( yo yo 0 ) = k r ^ ( X 2 + y 2 yo 0 )

dónde r ^ actúa en la dirección de la flecha roja, es decir r ^ = ( porque θ , pecado θ ) . El X y y por lo tanto, los componentes son

F X = k porque θ ( X 2 + y 2 yo 0 ) F y = k pecado θ ( X 2 + y 2 yo 0 )

dónde

θ = arcán ( y X )

Si tuviéramos que seguir un método similar al suyo, obtendríamos

F X = k ( X yo 0 ) F y = k y

Lo cual es incorrecto y corresponde al caso de dos resortes independientes con constantes idénticas que actúan sobre la masa.

Tomemos el caso con dos resortes idénticos:

ingrese la descripción de la imagen aquí

Con base en el análisis anterior, puede ver fácilmente que

F 1 = k r ^ 1 ( X 2 + y 2 yo 0 ) F 2 = k r ^ 2 ( ( X L ) 2 + y 2 yo 0 )

dónde r ^ 1 = ( porque θ 1 , pecado θ 1 ) y r ^ 2 = ( porque θ 2 , pecado θ 2 ) . De ello se deduce que el X , y los componentes son

F X = k porque θ 1 ( X 2 + y 2 yo 0 ) + k porque θ 2 ( ( X L ) 2 + y 2 yo 0 ) F y = k pecado θ 1 ( X 2 + y 2 yo 0 ) k pecado θ 2 ( ( X L ) 2 + y 2 yo 0 )

dónde

θ 1 = arcán ( y X ) θ 2 = arcán ( L X y )

Por lo tanto, las ecuaciones de los movimientos son bastante complicadas de resolver con exactitud. Si puede codificar, sugeriría resolverlos con algún integrador como Velocity Verlet .

Eso es exactamente lo que hizo floris en esta publicación: physics.stackexchange.com/questions/231364/… . Para ángulos pequeños esto se aproxima a Fx=-2kx y Fy=-2T/L. Si lo escribe, encontrará que los términos de orden superior ignorados son muy pequeños para la fuerza de alta tensión T, incluso en desplazamientos relativamente altos, como x=y=L/2.
@rickboender No había visto eso, gracias por la referencia. Puedo extender la respuesta actual para incluir una discusión sobre amplitudes pequeñas. De todos modos, la pregunta de OP no se trata específicamente de amplitudes pequeñas (incluso si el video muestra el caso de amplitudes pequeñas).
@rickboender Sería interesante ver si un análisis de pequeña amplitud puede dar el cambio de fase que OP dice que deberíamos obtener, pero sospecho que no...
Tampoco creo que se pueda mostrar el cambio de fase que OP espera, las frecuencias son diferentes, por lo que en realidad no hay ningún cambio de fase. En cuanto a los pequeños desplazamientos, suele ser la única forma de mantener manejables este tipo de problemas. En los sistemas no lineales, la frecuencia y la amortiguación dependen del esfuerzo, por lo que cambian todo el tiempo y necesita un equipo de medición preciso para registrar todo el movimiento.

El cálculo de la rigidez en y la dirección es incorrecta. La rigidez en y -la dirección no depende de la rigidez del resorte k , pero sólo en la fuerza de tensión T en los resortes, y la longitud de los resortes en reposo (unidos a la masa).

La rigidez en y -la dirección viene dada por:

F = T pecado θ = T   y L

Dónde θ es el ángulo entre el resorte y los ejes X. Tenga en cuenta que el último signo '=' solo es válido para pequeños desplazamientos, pero también lo es todo el análisis. Si θ aumenta, el efecto de T disminuye con porque θ , y el efecto de la rigidez del resorte aumenta con pecado θ .

Normalmente no hay cambio de fase, porque las frecuencias en X y y las direcciones son diferentes. Sólo pueden coincidir con ciertos valores de k , longitud L de primavera y T . Si coinciden, el cambio se puede determinar con las condiciones iniciales, como lo hizo usted.

Si los resortes están en tensión T en equilibrio, y las amplitudes son pequeñas, entonces las fuerzas restauradoras son F X 2 k X y F y 2 T L y , dónde L es la longitud estirada del resorte, como se explica en Comprensión de la oscilación transversal en sistemas de 1 masa, 2 resortes . Si la longitud natural de los resortes es L 0 entonces T = k ( L L 0 ) entonces F y 2 k ( 1 L 0 L ) . Las oscilaciones en el X y y las direcciones son aprox. lineales e independientes, por lo que son armónicos simples, pero las frecuencias difieren en la relación F y / F X 1 L 0 L .

Esta diferencia de frecuencia significa que la diferencia de fase entre el X y y oscilaciones aumenta gradualmente. El movimiento no es como ninguno de sus gráficos, que muestran una diferencia de fase constante. En cambio, el movimiento pasa de una oscilación lineal como en el gráfico 1 a una oscilación elíptica en el gráfico 2 que se vuelve circular. Luego vuelve a ser elíptica, pero esta vez con la línea (=eje de la elipse) reflejada en el eje y. Después de volverse lineal nuevamente, la dirección de la oscilación se invierte y el ciclo comienza nuevamente. Este movimiento se ilustra con una animación en ¿Por qué la vibración en mi cable actúa de manera tan extraña? y también se puede ver en tu video.

Usando los datos que proporcionó ( L 0 = 13 C metro , L = 25 C metro ) , entonces 1 L 0 L 0.48 . Entonces las frecuencias deben estar en la razón F y / F X 0,69 . De las 2 ejecuciones en la primera mitad de su video, dentro de aproximadamente 8 segundos hay 11 ciclos del X oscilación y 7 ciclos de la y oscilación, por lo que F y / F X = 7 / 11 0,64 , que está razonablemente cerca de la predicción.

sin embargo, el X y y las oscilaciones no parecen ser independientes entre sí. En las 2 carreras en la segunda mitad del video, en el que X y y los movimientos ocurren al mismo tiempo, la razón F y / F X es aprox. 8 / 9 en lugar de 7 / 11 cuando estos movimientos son separados. La diferencia de frecuencia es significativamente menor y cada uno se ha movido hacia el otro. Hay dos razones para esto: (i) la aproximación de pequeña amplitud no se cumple, por lo que F X , F y cada uno depende de X y y - es decir, están acoplados; (ii) la energía también se acopla mediante fricción o histéresis. (Para ver un ejemplo de acoplamiento por fricción de dos movimientos independientes, consulte Física rotacional de un naipe ).

No es obvio cómo el π / 2 podría surgir la diferencia de fase sugerida por su profesor. Si una oscilación impulsara a la otra, conduciría por π / 2 . Eso podría suceder si hubiera dos masas acopladas, siendo una mucho más pesada que la otra. Pero aquí las masas son las mismas.

Del vídeo, F X 11 / 8 1.4 H z . A partir de sus medidas, asumiendo que k se relaciona con uno de los dos resortes, entonces F X 1 2 π 2 k metro = 1 2 π 2 × 60 , 000 570 2.3 H z . Posiblemente su valor de k ¿no es correcto?


La ecuación que dedujiste para F y solo se aplica cuando L L 0 . Entonces F y / F X 1 por lo que la diferencia de fase permanece aprox. constante. Si inicia el sistema desde el reposo, la diferencia de fase es cero (gráfico 1) porque ambos X y y comiencen en el desplazamiento máximo para que tengan la misma fase. Las amplitudes no tienen que ser las mismas, porque la frecuencia es independiente de la amplitud. Para tener una diferencia de fase constante como en el gráfico 2, puede empujar la masa en el X o y dirección a medida que lo suelta.

Si la amplitud de las oscilaciones se vuelve "grande", entonces la X y y las oscilaciones se vuelven no lineales y acopladas. Si hay muy poca o ninguna tensión en los resortes en equilibrio, entonces las oscilaciones transversales son no lineales incluso para amplitudes pequeñas, con una fuerza restauradora proporcional a y 3 .